What is Improper integral: Definition and 238 Discussions

In mathematical analysis, an improper integral is the limit of a definite integral as an endpoint of the interval(s) of integration approaches either a specified real number,






{\displaystyle \infty }
,







{\displaystyle -\infty }
, or in some instances as both endpoints approach limits. Such an integral is often written symbolically just like a standard definite integral, in some cases with infinity as a limit of integration.
Specifically, an improper integral is a limit of the form:





lim

b







a


b


f
(
x
)

d
x
,


lim

a








a


b


f
(
x
)

d
x
,


{\displaystyle \lim _{b\to \infty }\int _{a}^{b}f(x)\,dx,\qquad \lim _{a\to -\infty }\int _{a}^{b}f(x)\,dx,}
or





lim

c


b









a


c


f
(
x
)

d
x
,


lim

c


a

+







c


b


f
(
x
)

d
x
,


{\displaystyle \lim _{c\to b^{-}}\int _{a}^{c}f(x)\,dx,\quad \lim _{c\to a^{+}}\int _{c}^{b}f(x)\,dx,}
in which one takes a limit in one or the other (or sometimes both) endpoints (Apostol 1967, §10.23).
By abuse of notation, improper integrals are often written symbolically just like standard definite integrals, perhaps with infinity among the limits of integration. When the definite integral exists (in the sense of either the Riemann integral or the more advanced Lebesgue integral), this ambiguity is resolved as both the proper and improper integral will coincide in value.
Often one is able to compute values for improper integrals, even when the function is not integrable in the conventional sense (as a Riemann integral, for instance) because of a singularity in the function or because one of the bounds of integration is infinite.

View More On Wikipedia.org
  1. G

    Integration by Parts: Improper Integral

    I used u substitution to get to this point: \lim_{R\rightarrow\ 3-}} \int_{-3}^{R-3} (\frac{u + 3}{sqrt{u}}) du is the only way to proceed from here using integration by parts?
  2. C

    Prove Convergence of Integral (1/x)sin(1/x)dx from 0 to 1

    Homework Statement Show that the integral of (1/x)sin(1/x)dx from 0 to 1 is converges absolutely? Homework Equations The Attempt at a Solution Should we use the comparison test in this situation?
  3. T

    A volume question with improper integral

    i added a link with the question and how i tried to solve it http://img167.imageshack.us/my.php?image=img8215vv0.jpg
  4. T

    How can substitutions in definite integrals affect the original variable?

    i added my question and how i tried to solve it in the link http://img208.imageshack.us/my.php?image=img8214ew9.jpg
  5. T

    Can You Tackle This Improper Integral Challenge?

    i added a link with my question and how i tried to solve it int the link http://img141.imageshack.us/my.php?image=img8212eq4.jpg
  6. F

    Evaluating Improper Integral: 6/(5x-2) from -∞ to 0

    Regarding the integral from -infinity to 0 of 6/(5x-2) --> I arrive at 6/5*ln(u), is this the right thing? How would I evaluate something like ln(-2), or do I just assume it is divergent since it the other limit will come out to neg infinity?
  7. F

    Calculate Improper Integral from 0 to Infinity: xe^(-5x)dx Solution

    For the integral from 0 to infinity of xe^(-5x)dx... I am getting as far as: -1/5*x*e^(-5x) + 1/5*int of (e^(-5x)dx) But I am getting stuck at this point. We are supposed to come out with 1/25 for the answer but how would I evaluate the "-1/5*x*e^(-5x)" since that is already out of...
  8. tony873004

    Solving Improper Integral: \int_0^{33} {\left( {x - 1} \right)^{ - 1/5} } \,dx

    evaluate \int_0^{33} {\left( {x - 1} \right)^{ - 1/5} } \,dx The back of the book gives 75/4 as the answer. I get 80/4. It has an asymptote at 1 since 0^(-1/5) DNE, and 0.00000001^(1/5) equals a number that grows larger as I add more 0's. But any time I try to raise a negative number...
  9. rocomath

    Where am i screwing up? Improper Integral

    the SM shows a different method but i figure i should arrive at the same answer either way. what step am i screwing up on? thanks! \int_{ - \infty}^{\infty}\frac {x^{2}dx}{9 + x^6} \int_{ - \infty}^{0}\frac {x^{2}dx}{9 + x^6} + \int_{0}^{\infty}\frac {x^{2}dx}{9 + x^6} u = x^{3} du =...
  10. F

    Improper integral concept help.

    Ok, my book has the example int from -1 to 2 of dx/x^3 this gets split into int from -1 to 0 of dx/x^3 and int from 0 to 2 of dx/x^3. Now, he had previously determined that the second integral returned positive infinity (diverges) by taking the lim as b approaches 0+. So, the book...
  11. E

    Convergence of Improper Integral in 3-Space

    Homework Statement Discuss the convergence of the integral 1/[x^2 + y^2 + z^2 + 1]^2 dxdydz in the whole space. Homework Equations The Attempt at a Solution Since the space is unbounded, the integral is an improper integral so we can consider a sphere with radius N and take...
  12. K

    Is My Solution to the Improper Integral Correct?

    Hie, I have attempted the following inproper intergral, and I just wanted to check that I have answered it the correctly, if not, any tips would be great. I attached them as pics. I attached my steps as pics, as I am not sure how to writ ein text on these forums. My answer is thus = 0...
  13. M

    Mastering Improper Integrals: Convergence & Divergence Techniques

    Hi, I'm having a bit of trouble showing that the integral from 1 to infinity of x/((1+x^6))^1/2 converges or diverges by the comparative property. I'm not sure if I'm setting it up right, but so far I have that 1/rad(1+x^6) is less than or equal to x/rad(1+x^6) which is less than 1/rad(x^6)...
  14. T

    Solve Improper Integral: $\int\frac{dx}{x\sqrt{x^2-4}}

    \int \frac{dx}{x \sqrt{x^2-4}} there are bounds to this problem but it is irrelevant since my problem is with the integration and not finding the limits. this integral resembles that of arcsec(x) but I am not sure how to deal with the -4. is there any way to solve this with partial...
  15. M

    Improper Integral Question/Check

    Hello, I just finished up two problems for my homework and I have a sneaking notion that I have made a mistake somewhere because when I checked the answer numerically by calculator and I get a differing number. I'm doing improper integrals for my real analysis class and the problem is stated...
  16. J

    Solving ln|x| Improper Integral: -1 to 1

    hi, why is ln|x|, from -1 to 1, converging? is "0" the bad point, and must i break up the integral from 1 to $, where $ = 0, and from $ to -1... so i have xlnx-x as my derivative... and i get -2? thanks
  17. S

    Understanding Improper Integrals: Solving for lim(t-->-inf) (1/3)e^(-t^3)=0

    Why does lim(t-->-infinit) (1/3)e^(-t^3)=0? or is it -infinit? I'm lost.
  18. S

    Solving Improper Integral: X/(sqrt(1+x^6) dx

    basically I'm stuck with this problem... Integral the upper limit is positive infinite and the lower limit is 1 X/(sqrt(1+x^6) dx... can someone give me an idea on how to start this?...I really don't know. Thanks.
  19. L

    Improper integral and rectangle method

    Improper integral and "rectangle" method If we have a definite integral then..using "rectangle" method we can get the approximation: \int_{a}^{b}f(x)dx \sim \sum_{n=0}^{N}f(a+nh)h My question is..how do you define this method when b-->oo (Imporper integral?)...:confused: :confused:
  20. S

    Solving Improper Integral: \int \frac{dx}{(ax+b)^2}

    Hi, for some reason I can't remember and I've been looking everywhere for some info but can't find anything. I am trying to find out the answer to : \int_1^i^n^f^i^n^i^t^e \frac{1}{(3x+1)^2}dx what is the integral of \int \frac{dx}{(ax+b)^2}? Thanks
  21. G

    Cauchy P.V. of an Improper Integral

    I was doing a Fourier Transform Integral, and was wondering if it would be legitimate for me to choose a semicircle CR on the lower half-plane below the real axis rather than choosing a semicircle CR on the upper half-plane above the real axis. I would expect it to be valid because the contour...
  22. G01

    Solving Integral: $\int_{-\infty}^{\infty} e^{-|x|} dx$

    \int_{-\infty}^{\infty} e^{-|x|} dx Could someone tell me why this integral, when you split it comes out to be: \int_{-\infty}^0 e^x dx + \int_0^{\infty} e^{-x} dx I keep thinking it should be e^(-x) in the first integral. I don't know why its positive. I can solve this integral...
  23. R

    Improper integral of 1/x from –1 to 1

    I am a physicist, not a mathematician. This problem has bothered me for 40 years. All introductory Calculus texts would consider this integral divergent. An example found in many texts is the integral of 1/(x-2) from 0 to 3, which is just a variant to the question I am asking. What I find...
  24. N

    Can You Solve This Improper Integral Trick for [0, infinity]?

    Int[(x^3)/((e^x)-1)] [0, infinity] What is the trick? I thought of by-parts but how would you integrate 1/((e^x)-1)? Substitution won't work with u = e^x -1 with x = ln|u+1| or it would be rather tough to evaluate u^3 Someone please give me a hint.
  25. T

    Improper integral using the Residue Theorem

    I'm trying to find \int\limits_0^{ + \infty } {\frac{{\sin x}} {{x^3 + x}}dx} Since the function is even, I can compute it as \frac{1} {2}\int\limits_{ - \infty }^{ + \infty } {\frac{{\sin x}} {{x^3 + x}}dx} To use the residue theorem, I construct a large semi-circle C with center O and...
  26. A

    Why Does Integral 1/x dx from -1 to 1 Diverge?

    Can someone explain why the following improper integral diverges? Integral 1/x dx from -1 to 1 I know if you break it up the individual integrals (from -1 to 0 and 0 to 1) diverge to negative infinity and infinity, whose sum is indeterminant in general, but the symmetry of the integral...
  27. M

    Improper integral + Maclaurin series problem

    Can you please offer any hints or suggestions on how to do these two problems: 1) Find the Maclaurin series of (x^2 + 1)/(3x^2 + 2x - 1). Should I perform long-division first? I can't seem to find any repeating pattern... 2) Evaluate the integral sqrt(12-4x-x^2) from x=2 to x=6. I...
  28. P

    Convergence of Improper Integral

    I have to analize the convergence of the following integral: \int_0^1 \frac {x^2+1} {\sqrt x * (1-x)^{5/4}} I tried to divide it between 0-1/2 and 1/2-1 and on the first one i reached to: \int_0^{1/2} \frac {x^2+1} {\sqrt x * (1-x)^{5/4}}<=\int_0^{1/2} \frac {x^2+1} {x^{14/4}} can i say...
  29. C

    How to solve an Improper Integral of Type 2?

    I've posted on Homeworks one of the number I did not understand. However, I would like to know the steps to calculate an improper integral of type 2. The type 2 is the one from constant a to constat b, not the one with inifnite. Please tell me the steps the accomplish it. :smile: I know...
  30. T

    Can l'Hôpital's Rule Help Determine the Limit of an Improper Integral?

    Proving an indeterminate form Prove for all positive integers n that \lim_{x\rightarrow 0}x({lnx})^n=0 Thanks for any help.
  31. N

    Evaluating Improper Integral of $\frac{x\arctan{x}}{(1+x^2)^2}$

    Evaluate the integral: \int^{\infty}_{0}\frac{x\arctan{x}}{(1+x^2)^2}dx can anybody give me some hint? :cry: Thanks in advance
  32. Galileo

    Exchanging derivative and improper integral

    I was wondering. When is the following legal? \frac{\partial}{\partial y}\int_{-\infty}^{\+\infty}f(x,y)dx=\int_{-\infty}^{\+\infty}\frac{\partial f(x,y)}{\partial y}dx I know the rule when the limits of integration are bounded, but here there are four limits involved. One for the...
  33. J

    Does this improper integral converge?

    Does this improper integral converge? I have an interesting problem that has been bothering me. Given: f(x)=1/x g(x)= (any continuous, non-negative function) g(x)<f(x) A = (a positive constant) I want to know: Does the integral of g(x) from A to +infinity converge? Or...
  34. C

    Find Function f(x) to Satisfy Lim & Improper Integral

    I need to find a function f(x), if one exists, such that: lim (x->inf) x^2*f(x) = 0 And the improper integral of f(x) from 1 to infinity doesn't exist. I'm thinking that no function can satisfy these requirements, but apparently I'm wrong... help anyone?
  35. M

    Infinite and improper integral help.

    I can't seem to evaluate integrals with infinite limits as well as improper integrals. Can anyone help in that? Sorry if this is a little vague but I'm stumped by the whole topic !
  36. F

    Calculating Improper Integral of x^3/(x^4-3)^1/2

    I stopped at the last step while calculating this improper integral: integral of x^3\ ( x^4 - 3)^1\2 with limits from 1 to infinity... that's x cubed over the square root of x raised to 4 minus 3... after replacing infinity with b and taking the limits it seems that I have to take...
  37. S

    What is the solution to the improper integral of 1/x?

    OK, so I'm trying to work out this: \int^{\infty}_a \frac{\dx}{x} Where a is a positive constant. Can you evaluate this analytically? I'm thinking the limit must exist, but \ln \left( \infty \right) = \infty , or at least tends to it in the limit. So can someone tell me the deal...
  38. tandoorichicken

    Solving Improper Integrals: \int_{3}^{6}, \lim_{t\rightarrow\infty}

    How do I do \int_{3}^{6} (5-x)^{\frac{-1}{3}} \,dx ? I also need to know how to do \lim_{t\rightarrow\infty} \int_{-t}^{t} \frac{x}{x^2 + 1} \,dx
Back
Top